Last visit was: 17 May 2024, 22:00 It is currently 17 May 2024, 22:00
Close
GMAT Club Daily Prep
Thank you for using the timer - this advanced tool can estimate your performance and suggest more practice questions. We have subscribed you to Daily Prep Questions via email.

Customized
for You

we will pick new questions that match your level based on your Timer History

Track
Your Progress

every week, we’ll send you an estimated GMAT score based on your performance

Practice
Pays

we will pick new questions that match your level based on your Timer History
Not interested in getting valuable practice questions and articles delivered to your email? No problem, unsubscribe here.
Close
Request Expert Reply
Confirm Cancel
SORT BY:
Date
Manhattan GMAT Online Marketing Associate
Joined: 14 Nov 2013
Posts: 272
Own Kudos [?]: 51 [0]
Given Kudos: 0
Send PM
Manhattan GMAT Online Marketing Associate
Joined: 14 Nov 2013
Posts: 272
Own Kudos [?]: 51 [0]
Given Kudos: 0
Send PM
Manhattan GMAT Online Marketing Associate
Joined: 14 Nov 2013
Posts: 272
Own Kudos [?]: 51 [0]
Given Kudos: 0
Send PM
Manhattan GMAT Online Marketing Associate
Joined: 14 Nov 2013
Posts: 272
Own Kudos [?]: 51 [1]
Given Kudos: 0
Send PM
The Last Two Weeks Before Your GMAT, Part 1: Building Your Game Plan [#permalink]
1
Bookmarks
Expert Reply
FROM Manhattan GMAT Blog: The Last Two Weeks Before Your GMAT, Part 1: Building Your Game Plan


Did you know that you can attend the first session of any of our online or in-person GMAT courses absolutely free? We’re not kidding! Check out our upcoming courses here.

What’s the optimal way to spend your last two weeks before the GMAT? Several students have asked me this question recently, so that’s what we’re going to discuss today! There are two levels to this discussion: building a Game Plan and how to review. We’ll discuss the former topic in the first half of this article and the latter in the second half.

What is a Game Plan?
For the past several months, you’ve been focused on lifting your score. During the last two weeks before your test, your focus needs to shift: your skills are what they are and your score is what it is. These things are not going to change an enormous amount in the last two weeks.

What could happen, though, is that your score actually drops on test day because you spent the last couple of weeks trying to build up a few weaknesses and you forgot a bunch of strategies that you last reviewed five weeks ago. You can tweak some weaknesses in the final two weeks, but now you need to focus on the big picture.

(If your immediate reaction to this is, “But my score is nowhere near where I want it to be!!” then be smart and postpone your test. You’re not going to have a huge score increase in just two weeks.)

Your Game Plan will help you to make certain decisions quickly during the test. When is it a good idea to spend an extra 20 or 30 seconds on a problem? When should you decide to make an educated guess? When should you cut yourself off completely, guess immediately, and move on? What should you do if you find yourself ahead or behind on your timing? (We’ll discuss the answers to these questions later in the article.)

Your Game Plan will also help you to prioritize your review based upon your strengths and weaknesses. You’re going to review your major strategies in all areas, the major content you need to know (don’t try to cram everything into your brain; review the stuff that shows up the most!), your pacing, your educated guessing strategies, and so on. As you do that, the data you gather will help you to tweak your game plan further.

Building Your Game Plan
Your Game Plan is a dynamic thing. You perfect it a little bit more every few days as you gather more data and continue to review.

What Does My Gut Say?

First, make a list of your major strengths and weaknesses. Start with the five question types, but also drill down further into specific content areas (Quant, SC) and question sub-types (CR, RC). Consider both accuracy and timing when assessing your strengths and weaknesses; the two factors are equally important.

What Does The Data Say?

We can’t rely only on our gut feelings to know our strengths and weaknesses. Our gut is often right, but it is sometimes wrong—more often than most people realize. It’s also important to check your data.

Here’s how to analyze your practice CATs.

If you have been tracking your accuracy and timing on OG problems, also examine that data. (Note: if you are using our GMAT NavigatorTM program, you can view the data reports there.)

Split individual OG question types (DS, PS, CR, RC, SC) into three broad groups: the first third in that section in the book (the generally easier questions), the middle third (the medium questions), and the final third (the generally harder questions). This will give you an idea of how your performance is changing as the questions get harder.

Note: if you’re early in the whole study process right now, I highly recommend tracking your work on the OG problems. Create a simple spreadsheet and keep track of the specific source (book), question number, time spent the first time you did it, and whether you got it right or wrong. You can also add notes about what you want to learn, memorize, review, or do, based upon each problem. (If you do have access to GMAT Navigator, use that too!)

How to Use Your Game Plan
Generally, it’s a good choice to spend an extra 20 to 30 seconds when a problem is a strength for you, and only then when you know exactly what to do but the problem is on the harder side and so needs a little extra time. Also, note that I said “20 to 30 seconds” above. Even if something is a strength, spending an extra minute or more pretty much guarantees at least one other question wrong on the test due to rushing, careless mistakes, or running out of time at the end.

Do not spend extra time on weaknesses (you can spend normal time, just not extra time). That may sound like common sense, but when we’re in the middle of the test, we’re often reluctant to let go of our weaker problems. If you know what your weaknesses are, you can let those problems go more easily—after all, you know it’s a weakness so you know there’s less chance you’re going to get it right. Get it wrong before you lose any time so that you don’t make the situation even harder for yourself.

If you suddenly realize that you have been on one problem for an awfully long time—you’re not even sure how long—stop yourself immediately, guess, and move on. Suppress the urge to think that you can get it right if you just spend a little more time. This is especially true if you are already behind on time.

If you realize that you are ahead or behind on timing at any point during the test, take steps to correct the situation right away. Do not think that the problem will fix itself (it won’t!) and don’t underestimate the dangers of being too far behind or too far ahead. Generally, if you’re within 2 to 3 minutes of your pacing plan, you’re fine. If you are off by more than that, take action.

If you are moving too quickly, make yourself start writing everything down. Take notes. Write down all calculations. Track the answers on your scrap paper. Basically, you need to be more systematic to ensure that you are not losing points to careless errors due to speed.

If you are moving too slowly, use your Game Plan. As soon as you see a problem that’s an area of weakness for you, guess immediately and move on. Sacrifice that problem to gain a significant amount of time back. If that’s not enough to catch you up, do it again the next time you see a “weakness” problem.

How To Practice Your Game Plan
About 10 to 14 days before the test (ideally closer to 14), review your Game Plan and take a practice test under official conditions, including the essay and IR sections, the lengths of the breaks, and so on. Practice implementing your Game Plan during that test. Then review the test with an eye toward improving your Game Plan. Where did you make good decisions about how to spend your time or how to handle a certain problem? Where did you make poor decisions? What should you have done instead? How are you going to make sure that you make the right decision next time? Figure out ahead of time how you’re going to handle different kinds of situations. Then, on the test, you don’t have to think about what to do; you can just react.

Spend the next 5 to 7 days practicing and refining your Game Plan on shorter sets of questions. Intersperse this with your general review of content, question types, and so on. Then, about 5 to 7 days before the test (ideally closer to 7), take another practice test under official conditions. Practice implementing your Game Plan again, then go through your analysis, and refine further.

Finally, implement your Game Plan on test day! And don’t forget to join us next time, when we’ll talk about how to Review during the last 14 days

Take-Aways
1) Change your focus during the final two weeks of study: away from learning new stuff, and toward reviewing material and developing your Game Plan.

2) Practice and refine your Game Plan over the last two weeks.

3) Use your Game Plan on test day!

Can’t get enough of Stacey’s GMAT expertise? Attend the first session of one of her upcoming GMAT courses absolutely free, no strings attached. Seriously. 


Stacey Koprince is a Manhattan Prep instructor based in Montreal, Canada and Los Angeles, California. Stacey has been teaching the GMAT, GRE, and LSAT  for more than 15 years and is one of the most well-known instructors in the industry. Stacey loves to teach and is absolutely fascinated by standardized tests. Check out Stacey’s upcoming GMAT courses here.

The post The Last Two Weeks Before Your GMAT, Part 1: Building Your Game Plan appeared first on GMAT.
This Blog post was imported into the forum automatically. We hope you found it helpful. Please use the Kudos button if you did, or please PM/DM me if you found it disruptive and I will take care of it. -BB
Manhattan GMAT Online Marketing Associate
Joined: 14 Nov 2013
Posts: 272
Own Kudos [?]: 51 [0]
Given Kudos: 0
Send PM
Wharton Team-Based Discussion 2017: What to Expect and How to Prepare [#permalink]
Expert Reply
FROM Manhattan GMAT Blog: Wharton Team-Based Discussion 2017: What to Expect and How to Prepare


Don’t be nervous about your team-based interview at Wharton—our friends at mbaMission are offering important tips and hosting a team-based discussion simulation for Wharton hopefuls!

The Wharton School at the University of Pennsylvania plans to send out Round 2 interview invitations on February 8, and once again the school is using its team-based discussion format to evaluate MBA candidates, in place of a traditional business school admissions interview. Understandably, Wharton applicants get anxious about this atypical interview, because the approach creates a very different dynamic from what one usually encounters in a one-on-one meeting—and with other applicants also in the room, one cannot help but feel less in control of the content and direction of the conversation. Yet despite the uncertainty, here are a few things that interviewees can expect:

  • You will need to arrive at the interview with an idea—a response to a challenge that will be presented in your interview invitation.
  • Having the best idea is much less important than how you interact with others in the group and communicate your thoughts. So while you should prepare an idea ahead of time, that is only part of what you will be evaluated on.
  • Your peers will have prepared their ideas as well. Chances are that ideas will be raised that you know little or nothing about. Do not worry! The admissions committee members are not measuring your topical expertise. Instead, they want to see how you add to the collective output of the team.
  • After the team-based discussion, you will have a short one-on-one session with someone representing Wharton’s admissions team. More than likely, you will be asked to reflect on how the team-based discussion went for you; this will require self-awareness on your part.
To give candidates the opportunity to undergo a realistic test run before experiencing the actual event, we created ourTeam-Based Discussion Simulation. Via this simulation, applicants participate anonymously with three to five other MBA candidates in an online conversation, which is moderated by two of our experienced Senior Consultants familiar with Wharton’s format and approach. All participants then receive feedback on their performance, with special focus on their interpersonal skills and communication abilities. The simulation builds confidence by highlighting your role in a team, examining how you communicate your ideas to—and within—a group of (equally talented) peers, and discovering how you react when you are thrown “in the deep end” and have to swim. Our Wharton Team-Based Discussion Simulation allows you to test the experience so you will be ready for the real thing! 

The 2017 Wharton Team-Based Discussion Simulation schedule is as follows:

  • Group A: Sunday, February 12 at 10:00 AM ET
  • Group B: Sunday, February 12 at 12:00 PM ET
  • Group C: Monday, February 13 at 9:00 PM ET
  • Group D: Tuesday, February 14 at 6:00 PM ET
  • Group E: Wednesday, February 15 at 6:00 PM ET
  • Group F: Friday, February 17 at 4:00 PM ET
  • Group G: Sunday, February 19 at 10:00 AM ET
  • Group H: Sunday, February 19 at 12:00 PM ET
  • Group I: Tuesday, February 21 at 9:00 PM ET
  • Group J: Wednesday, February 22 at 9:00 PM ET
  • Group K: Friday, February 24 at 4:00 PM ET
  • [b]Group L: Sunday, February 26 at 12:00 PM ET[/b]


To learn more or sign up for a session, visit ourWharton Team-Based Discussion Simulation page.

mbaMission is the leader in MBA admissions consulting with a full-time and comprehensively trained staff of consultants, all with profound communications and MBA experience. mbaMission has helped thousands of candidates fulfill their dream of attending prominent MBA programs around the world. Take your first step toward a more successful MBA application experience with a free 30-minute consultation with one of mbaMission’s senior consultants. Sign up today atwww.mbamission.com/manhattangmat.



The post Wharton Team-Based Discussion 2017: What to Expect and How to Prepare appeared first on GMAT.
This Blog post was imported into the forum automatically. We hope you found it helpful. Please use the Kudos button if you did, or please PM/DM me if you found it disruptive and I will take care of it. -BB
Manhattan GMAT Online Marketing Associate
Joined: 14 Nov 2013
Posts: 272
Own Kudos [?]: 51 [0]
Given Kudos: 0
Send PM
The Knowledge Trove that Is a GMAT Ratio [#permalink]
Expert Reply
FROM Manhattan GMAT Blog: The Knowledge Trove that Is a GMAT Ratio


Did you know that you can attend the first session of any of our online or in-person GMAT courses absolutely free? We’re not kidding! Check out our upcoming courses here.

‘Ratios?’ You might be thinking. ‘Those are easy. Pretty sure I get those.’

Wait. Let’s pretend I am the eccentric owner of a pet store, and I sell only two types of pets: rabid donkeys and three-legged mountain lions. In my store the ratio of donkeys to lions is 3:7. What do you know?

‘….That for every 3 donkeys you have 7 lions. Thanks for all the information, weird guy, should I get an external hard-drive so I can back all that up?’

Okay, drop the sarcasm, reader of my invention. When the GMAT gives you a ratio, it actually contains a boatload of information. Take a second and brainstorm what else you can figure out about this pet store. Anything at all.

You didn’t do the exercise, but whatever, I’ll continue anyway.

How about at the simplest level: I have more lions than donkeys. That can be useful information!

How about this: what fraction/percent of my animals are donkeys? (Careful, if you say 3/7, you fell for the most common ratio trap. Ratios compare parts to parts. To get a fraction or percent of the total, we must first add these parts. My relative total is the sum of the relative parts: in this case 3+7=10).

So my fraction/percent that are donkeys: 3/10, or 30%. 70% of these animals are lion. So when I’m given a ratio of the parts, I can tell you percentages of the total. Also I can give you relative percents between animals themselves. I have 133% more lions than donkeys, which is the same as saying 233% the number of donkeys is my number of lions. The inverse of that: I have approximately 57% fewer donkeys than lions (4/7 is approximately 57%), which is the same as saying the number of donkeys I have is 43% the number of lions.

Note that we’re working from ratio to these percentages, but you can also work from the percentages to the ratio. When I’m given a relative value between items, I can typically tell you every relative value between them.

Another question: which of the following could be the number of donkeys in my pet store?

-82

-83

-84

-85

-86

Because we’re dealing with discrete (that is, countable in whole number) items (let’s pretend my pet store isn’t so eccentric as to sell half-donkeys), just by knowing the ratio, we know something about the possible number of donkeys. And lions. And the total number of animals.

Remember, whenever you have a ratio, you also have an ‘unknown multiplier’—some value that you can multiply each relative value of the ratio to get the actual values of those items. Let’s call the unknown multiplier ‘m’ (I avoid using x for the multiplier, because a lot of students will solve for ‘x’ and forget they aren’t done with the problem; the GMAT never asks for the multiplier itself, but they use it as a trap answer sometimes).

So the ratio in actual values is 3m:7m (which simplifies to the 3:7 we’re given).

Now let’s consider the possibilities for this ratio. The simplest case is we have 3 donkeys, 7 lions, and 10 total animals. That is, the unknown multiplier is 1. This… never happens on the GMAT. It’s a great idea to pick those values on a problem that can be solved with smart numbers, but otherwise, just realize the unknown multiplier is virtually never ‘1’. What other possibilities might we have?



etc. etc. etc.

What do you notice?

The donkeys are in multiples of 3, the lions in multiples of 7, and the total in multiples of 10. In the question above, the only choice that was a multiple of 3 is 84.

“BUT WAIT! THERE’S MORE!”

Okay, Billy Mays, calm down.

Which of the following could be the positive difference between the number of donkeys and the number of lions?

-42

-46

-50

-64

-66

Let’s look at the chart again, replacing the ‘total’ column with ‘difference’:



The unknown multiplier also applies to the difference between the parts of your ratio! 7-3=4, so our final difference must be a multiple of 4. The only answer that was a multiple of 4 was 64.

Also, we can use a ratio to set up an equation (useful in solving a system of equations). If the ratio of D:L is 3:7, then D/L = 3/7, or 7D=3L (this last one is the equation the GMAT can give you when it’s trying to hide that it’s showing you a ratio).

Your job is to recognize which facts are useful in the problem you’re solving, but in order to do that it’s important to understand what information is even available to you when the GMAT gives you what looks like a boring two or three number ratio.

Now. I’m going to get back to wiping foam off this donkey’s mouth. Happy studying.

Want some more GMAT tips from Reed? Attend the first session of one of his upcoming GMAT coursesabsolutely free, no strings attached. Seriously.

Reed Arnold
is a Manhattan Prep instructor based in New York, NY.
He has a B.A. in economics, philosophy, and mathematics and an M.S. in commerce, both from the University of Virginia. He enjoys writing, acting, Chipotle burritos, and teaching the GMAT. Check out Reed’s upcoming GMAT courses here.

The post The Knowledge Trove that Is a GMAT Ratio appeared first on GMAT.
This Blog post was imported into the forum automatically. We hope you found it helpful. Please use the Kudos button if you did, or please PM/DM me if you found it disruptive and I will take care of it. -BB
Manhattan GMAT Online Marketing Associate
Joined: 14 Nov 2013
Posts: 272
Own Kudos [?]: 51 [0]
Given Kudos: 0
Send PM
GMAT Grammar: Pronoun Rules [#permalink]
Expert Reply
FROM Manhattan GMAT Blog: GMAT Grammar: Pronoun Rules


Did you know that you can attend the first session of any of our online or in-person GMAT courses absolutely free? We’re not kidding! Check out our upcoming courses here.

Possessive Pronouns
Pronouns are nifty little tools for consolidating your writing. Instead of repeating a noun over and over within the same sentence, you can simply replace it with a pronoun. The meaning stays clear and the message is concise. Compare the following sentences:

When Arjun spilled the coffee, the coffee drenched all the papers that Arjun was working on and the coffee made the papers nearly illegible.

When Arjun spilled the coffee, it drenched all the papers that he was working on and it made them nearly illegible.

You understand the meaning in either case, but one sounds unnecessarily wordy. Conversations would be much longer if pronouns didn’t exist. So let’s get down to the nitty-gritty of what pronouns are and how they work.

A pronoun is a word that can replace a noun or a noun phrase. We use them all the time without even noticing. When you point to a soda and ask “Can you hand me that?” you’re using a pronoun.

They’re even simple in terms of the GMAT. You’ve got to keep in mind three things:

  •  The pronoun must have an antecedent (defined below).
  •  The pronoun and antecedent must agree in number.
  •  The pronoun must be unambiguous.
Let’s dive in to what these rules mean.

Rule 1: The pronoun must have an antecedent.
An “antecedent” is the noun or noun phrase that the pronoun refers to. If we revisit our example of coffee spilling, you should find three pronouns, each with its own antecedent. Try to spot all three pairs before reading on to the explanation.

When Arjun spilled the coffee, it drenched all the papers that he was working on and it made them nearly illegible.

The first pronoun in the sentence is “it.” “It” is the thing that’s drenching the papers, so we can reason that “it” must be the coffee. “Coffee” is the antecedent to “it.” Hopefully you were able to pair the other two as well: “he” and “Arjun”, “them” and “all the papers.”

This rule is actually a little more strict on the GMAT than in standard English. Typically, you’re not reading a sentence on its own, you’re reading it in the context of a paragraph or a larger article. Therefore, it’s entirely acceptable to write a sentence with a pronoun referring to an antecedent in a different sentence. But on Sentence Correction questions, we’re restricted to a single sentence, so the antecedent must actually show up in the same sentence.

See whether you can spot the error here:

Without realizing her mistake, Ana continued to overcook the squash, finally serving a dinner that they were not satisfied with.

Did you think the misbehaving pronoun was “her”? If so, you were wrong. “Her” has an antecedent: Ana. The fact that the noun comes after the pronoun is inconsequential, as long as the relationship is clear.

The actual culprit was “they.” Who’s they? The family members? The customers in Ana’s restaurant? The Ambassador to New Guinea and his wife? Based on this sentence, we have no idea. The antecedent is not present. If you missed this, you probably didn’t even realize a pronoun was present.

The number one reason why test takers choose answers that violate this first pronoun rule is that they don’t see the pronoun. So be on the lookout! Practice recognizing pronouns in emails you send, books you read, even ads on buses. In particular, be sure to pause every time you see any of the five most commonly tested pronouns:

It

Its

They

Them

Their

Once you spot it, verify that it has an antecedent. (Btw, did you notice my sloppy use of “it” in the previous sentence? There’s definitely no antecedent within that sentence, but you have the benefit of reading this in context, so you should know what “it” is.)

Rule 2: The pronoun and antecedent must agree in number.
If you consider the five common pronouns listed above, there’s a significant difference between the first two and final three. “It” and “its” replace singular antecedent, while “they,” “them,” and “their” all replace plural antecedents. Simple illustrations:

The watermelon is delicious and Sheri just can’t get enough of it.

Jorge was able to juggle three balls at a time, but only for about a minute before he dropped them.

So while you’re checking that the antecedent is present within the sentence, check that it actually agrees with the pronoun as well.

Rule 3: The pronoun must be unambiguous.
This is the hardest rule to enforce. Sometimes things sound ambiguous when they’re not. Sometimes you think they’re fine when there’s a major problem. Test yourself to see if you can spot the presence (or absence) of ambiguity:

All the people in the room gasped when Jacob and Hans revealed their evil plot.

Boots, including knee-highs and ankle-highs, come into fashion during colder months when they are a practical choice.

I’ll give you a minute to think…

Still thinking…

Ok, here’s the likely GMAT interpretation of the ambiguity. The first sentence is ambiguous, the second one is not.

Wait, what?

Nope, that’s not a mistake. The only pronoun in the first sentence is “their” and the meaning the author intended (speaking as the author) was not the revelation of Jacob’s and Hans’s evil plot. Consider who else “their” could refer to. It’s plural, so it needs to be a plural noun or noun phrase, and there is another candidate present. All the people! The people in the room were shocked when Jacob and Hans betrayed them by revealing the group’s evil plot. Such betrayal!

How could you know that my intended meaning was a vast group conspiracy rather than just a pair of troublemakers? You couldn’t. I gave no hints to that meaning in my sentence. But you also couldn’t know that wasn’t my intended meaning. The two plural nouns are both decent candidates for antecedents, so the sentence is ambiguous. If I change it to:

All the people in the room gasped when Jacob and Hans revealed the people’s evil plot.

then we would have no ambiguity. I’ve removed the ambiguity by replacing the pronoun with its antecedent. It’s longer, but clearer.

“But wait, Emily!” I hear you saying. If the issue is just that you need only one plural noun to go with the plural antecedent, isn’t the second sentence (reproduced below) also ambiguous?

Boots, which include both knee-highs and ankle-highs, come into fashion during colder months when they are a practical choice.

We have three plural nouns: boots, knee-highs, and ankle-highs. Couldn’t “they” refer to any of them and doesn’t that make this sentence ambiguous? I’m so glad you asked.

The answer is no. The two types of boots are being used as a descriptor (go back to our noun modifier post for a refresher on that if you’d like). “Boots” is the subject of the sentence, and “they” is the subject of a dependent clause (when they are a practical choice). You could reasonably match up two subjects, but it’s unlikely you’d match a subject with a random noun within a modifier.

We’re getting very much into the nuances of grammar, so let’s wrap up with how you’re going to figure this out with a time constraint on the GMAT.

The short answer is: you’re not. You’re going to use the structure of the GMAT to cheat. Well, not exactly cheat, but definitely to reduce unnecessary work. In your first glance, imagine you recognize that the endings of the five answer choices are:

A) … than they were

B) … than they are

C) … than it is

D) … then it was

E) … than it was

All of these answers have a pronoun. You don’t have the option of changing it back to a more clear, but less concise, noun. So we’re stuck with it. Go figure out if the antecedent is singular or plural, and eliminate accordingly. (Bonus points if you caught that D has an additional red flag.)

But in the situation that the ending of the answer choices are:

A) … than they were

B) … than they are

C) … than the books were

D) … then the books are

E) … than the books are

you need to go see if a pronoun is even reasonable. Does the presence of “they” introduce ambiguity?

It will take practice and patience to make the right determination, one of which you can find in our Sentence Correction Strategy Guide, the other you’ll need to cultivate on your own. And keep practicing spotting pronouns in everyday life. It’s an important skill for the GMAT! 

Of course, the most in-depth way to learn the ins-and-outs of the GMAT is to take a complete course with one of our master instructors. You can try out any first session for free! No strings attached. We promise.


Emily Madan is a Manhattan Prep instructor based in Philadelphia. Having scored in the 99th percentile of the GMAT (770) and LSAT (177), Emily is committed to helping others achieve their full potential. In the classroom, she loves bringing concepts to life and her greatest thrill is that moment when a complex topic suddenly becomes clear to her students. Check out Emily’s upcoming GMAT courses here. Your first class is always free!

The post GMAT Grammar: Pronoun Rules appeared first on GMAT.
This Blog post was imported into the forum automatically. We hope you found it helpful. Please use the Kudos button if you did, or please PM/DM me if you found it disruptive and I will take care of it. -BB
Manhattan GMAT Online Marketing Associate
Joined: 14 Nov 2013
Posts: 272
Own Kudos [?]: 51 [0]
Given Kudos: 0
Send PM
Navigating Proper MBA Interview Etiquette [#permalink]
Expert Reply
FROM Manhattan GMAT Blog: Navigating Proper MBA Interview Etiquette


Each week, we are featuring a series of MBA admission tips from the leading business school admissions consulting firm
mbaMission.

Making a good impression on your MBA admissions interviewer goes beyond simply answering questions well. Knowing how to dress properly and how to navigate certain etiquette issues is also crucial. You want to leave your interviewer confident that you have the maturity and professionalism necessary to succeed after graduation. The following are our answers to four questions that often bedevil applicants with respect to proper interview etiquette.

What should I wear?
Always follow any guidelines the school provides about proper interview dress. If business casual is specified, wear business casual; if business attire, dress in business attire. Jeans, T-shirts, and ripped or unclean clothing are never appropriate. If the school does not specify a dress code, wear business attire for any on-campus interviews as well as for an off-campus interview with a member of the admissions staff. Business casual is often best when meeting an alumnus/alumna off campus, though you may consider politely asking the person you are meeting about proper attire in advance. Showing some creativity and style with your clothing is okay, but do not go overboard—remember that your meeting is a professional one, and your first impression is vital.

I am meeting my interviewer at a coffee shop. Who pays?
If you are meeting an alumnus/alumna at a café or similar establishment for an interview, try to avoid the awkward “who pays?” scenario altogether by arriving a few minutes early, purchasing your own beverage, and then offering to pay for the interviewer’s selection when he/she arrives. If the person you are meeting arrives before you, you might politely offer to pay for his/her drink, but if the interviewer declines, you should not insist.

Should I send a thank you note?
Yes, you should always send a brief thank you note after your interview. Write and send the note as soon as possible after your meeting—the same day or the next is ideal—and be sure to mention specifics from your conversation or your visit. Emailing the thank you is fine. Interviewers usually need to submit their feedback on candidates within 24 hours, so you want your message to be received quickly.

If I am not sure how I did, can I ask for feedback?
No! Feeling anxious about how you performed is natural, but do not ask your interviewer for feedback. Doing so will not help establish you as professional or mature and will instead leave the impression that you lack good judgment (and confidence). Just be patient and wait for the admissions committee to make its decision!

mbaMission offers even more interview advice in their FREEInterview Primers, which are available for 16 top-ranked business schools. For even more individualized interview prep, try their one-of-a-kindMock Interview Sessions!


mbaMission is the leader in MBA admissions consulting with a full-time and comprehensively trained staff of consultants, all with profound communications and MBA experience. mbaMission has helped thousands of candidates fulfill their dream of attending prominent MBA programs around the world. Take your first step toward a more successful MBA application experience with a free 30-minute consultation with one of mbaMission’s senior consultants. Sign up today atwww.mbamission.com/manhattangmat.

The post Navigating Proper MBA Interview Etiquette appeared first on GMAT.
This Blog post was imported into the forum automatically. We hope you found it helpful. Please use the Kudos button if you did, or please PM/DM me if you found it disruptive and I will take care of it. -BB
Manhattan GMAT Online Marketing Associate
Joined: 14 Nov 2013
Posts: 272
Own Kudos [?]: 51 [0]
Given Kudos: 0
Send PM
Know the GMAT Code: Interest Rate GMAT Problems [#permalink]
Expert Reply
FROM Manhattan GMAT Blog: Know the GMAT Code: Interest Rate GMAT Problems


Did you know that you can attend the first session of any of our online or in-person GMAT courses absolutely free? We’re not kidding! Check out our upcoming courses here.

I’m excited about the problem I have to share with you today in the latest installment of our Know the Code series. ☺ Interest rate GMAT problems can be extremely annoying—you might find yourself spending 4 minutes and still having to guess in the end. So your first decision is whether you even want to tackle these kinds of problems in the first place.

But there are some things you can learn that could make answering interest rate GMAT problems a lot less irritating. Try out this Integrated Reasoning (IR) Two-Part problem from the GMATPrep® free practice exams. (Note: This one is an IR question, but I could absolutely see them testing the same principle on a Quant problem.)

If you’re planning to guess on 3 questions in the IR section, then you can give yourself 3 minutes and 20 seconds to do this problem. If you’re planning to guess on 2 questions, then give yourself 3 minutes.

“*Loan X has a principal of $10,000x and a yearly simple interest rate of 4%. Loan Y has a principal of $10,000y and a yearly simple interest rate of 8%. Loans X and Y will be consolidated to form Loan Z with a principal of $(10,000x + 10,000y) and a yearly simple interest rate of r%, where. In the table, select a value for x and a value for y corresponding to a yearly simple interest rate of 5% for the consolidated loan. Make only two selections, one in each column.”



Ready?



1-second Glance. Two-Part. Wall of words! Story—will need to translate.

Read and Jot. This one’s so complex that I needed to read it twice, and I didn’t start writing anything till the second time. Sometimes you need to do that with story problems.

There are two loans with some details and then a third one that’s a combination of the first two. Interesting. Let’s start with the first two.



Reflect. I’m also going to do a loop on my first two steps. I’m going to reflect a bit here, then continue with the rest of my second read-through and jot down the rest.

Hmm. If both of these were exactly $10k in principle, then combining them would give me a combined interest rate of 6%—the exact, or “straight,” average of the two interest rates. But the principles have these extra variables, x and y, and they probably don’t represent the same value—that would be too easy.

I have noticed one important thing, though: this problem is really a weighted average problem in disguise, with the x and the y representing the relative weights of the two original loans. The combined loan will depend on how much each of the original loans is weighted.

Read and Jot some more.



The first part is okay, but what is up with that weird formula for r? (I don’t know what it means, so I haven’t jotted it down yet.) And then that last bit—they’re telling me to calculate based on a simple interest rate of 5%…for the consolidated loan.

Hey! That’s Loan Z. They actually just told us that r% = 5%. ☺ Nice!

And here’s the even nicer thing: go back to that weird formula. Plug in r = 5.



That’s ugly. So make it less ugly. Simplify!



That’s certainly a much nicer equation. But what’s the significance? What is that telling us?

The question asks us to find a value for x and a value for y that correspond with all of the given information. This equation gives a relationship between those two variables. Whatever y is, multiply it by 3 to get x.

Go take a look at the possible answer choices. If y were 21, what would x have to be?

If y = 21, then x = (3)21 = 63. However, that value, 63, isn’t in the answers, so y doesn’t equal 21.

Try the next one. If y = 32, then x = (3)32 = 96. Bingo! That value is in the answers! The value for y is 32 and the value for x is 96. Done!

Now, wait a sec. What just happened here? How did that really work?

If you’re comfortable with the idea that the problem asked you for relative values of x and y, and all you really had to do was find that relative relationship and then find the two answers that fit that relationship, you’re good to go.

If, on the other hand, you want to understand the underlying principles here—and, by the way, if you’re interested in learning an even faster way to solve—then read on.

Remember, at the beginning, when I mentioned that this was a weighted average problem? We never followed up on that. Now we’re going to.

Loan X is 4% and Loan Y is 8%. And then they tell us the rate for the combined loan: it’s 5%. That’s really key!

If the combined loan rate is 5%, then we can figure out the relative proportion of Loan X to Loan Y using the teeter-totter method (we discuss this in the Weighted Averages chapter of our Word Problems Strategy Guide). And remember that Loan X = 10,000x and Loan Y = 10,000y. In other words, the relative values of x and y equal the relative weighting that each loan is given in the overall calculation.

Here’s how it works:



If the teeter totter were perfectly balanced, then the combined rate would be exactly halfway between Loan X and Loan Y, at 6%. It’s not perfectly balanced, though; it’s tilted over towards Loan X.

That leads to our first important conclusion: Loan X is more heavily represented, so the value of x is larger than the value of y. Keep that in mind if you get stuck and have to guess later.

Next, we can actually figure out the exact proportion of x to y. Here’s how:



There are two “sub-distances”: 5 − 4 = 1 and 8 − 5 = 3. The shorter one goes with the smaller loan, Y. The longer one goes with the larger loan, X. The values themselves represent the ratio of the two loans: x : y = 3 : 1. In other words, x is 3 times as large as Y.

That’s the same info that the earlier equation told us, and you can follow the same logic to get to the answer pairing 32 and 96. In other words, if you recognize that this is a weighted average, you can find the 3 : 1 ratio just by drawing a number line and doing some pretty basic subtraction. No algebra needed.

As I mentioned earlier, I can definitely see them using this same principle on a regular Quant question. The only major difference would be that IR questions do tend to provide more information than you need to answer a question, while Quant questions do not. So, in Quant-question form, the question stem would be streamlined: You’d be given only what you need in order to answer the question.

Key Takeaways for Knowing the Code
(1) Take long story problems in parts. You may need to read the whole thing first to understand the basic story, then read it a second time in order to jot down information and reflect on how to move forward.

(2) Don’t skip that Reflect step! In this case, there were two important keys to notice: first, that this is a weighted average problem in disguise, and second, that r = 5.

(3) Turn any knowledge you gain into Know the Code flash cards:



Happy studying! 

* GMATPrep® questions courtesy of the Graduate Management Admissions Council. Usage of this question does not imply endorsement by GMAC.

Can’t get enough of Stacey’s GMAT mastery? Attend the first session of one of her upcoming GMAT courses absolutely free, no strings attached. Seriously. 


Stacey Koprince is a Manhattan Prep instructor based in Montreal, Canada and Los Angeles, California. Stacey has been teaching the GMAT, GRE, and LSAT  for more than 15 years and is one of the most well-known instructors in the industry. Stacey loves to teach and is absolutely fascinated by standardized tests. Check out Stacey’s upcoming GMAT courses here.

The post Know the GMAT Code: Interest Rate GMAT Problems appeared first on GMAT.
This Blog post was imported into the forum automatically. We hope you found it helpful. Please use the Kudos button if you did, or please PM/DM me if you found it disruptive and I will take care of it. -BB
Manhattan GMAT Online Marketing Associate
Joined: 14 Nov 2013
Posts: 272
Own Kudos [?]: 51 [0]
Given Kudos: 0
Send PM
MBA Admissions Myths Destroyed: Business School Stereotypes [#permalink]
Expert Reply
FROM Manhattan GMAT Blog: MBA Admissions Myths Destroyed: Business School Stereotypes


What have you been told about applying to business school? With the advent of chat rooms, blogs and forums, armchair “experts” often unintentionally propagate MBA admissions myths, which can linger and undermine an applicant’s confidence. Some applicants are led to believe that schools want a specific “type” of candidate and expect certain GMAT scores and GPAs, for example. Others are led to believe that they need to know alumni from their target schools and/or get a letter of reference from the CEO of their firm in order to get in. In this series,mbaMission debunks these and other myths and strives to take the anxiety out of the admissions process.

Many business school applicants believe that the MBA admissions committees have distilled their criteria for selecting candidates over the years and have in mind a specific “type” of individual they want. For example, within this world of business school stereotypes, applicants believe that Harvard Business School (HBS) is looking only for leaders, Kellogg is looking only for marketing students, Chicago Booth is looking only for finance students, and even that MIT Sloan is looking only for “eggheads.” Of course, these business school stereotypes—like most stereotypes—are inaccurate. Chicago Booth wants far more than one-dimensional finance students in its classes, and it provides far more than just finance to its MBA students (including, to the surprise of many, an excellent marketing program). HBS is not a school just for “generals”; among the approximately 950 students in each of its classes, HBS has a wide variety of personalities, including some excellent foot soldiers. So, at mbaMission, we constantly strive to educate MBA candidates about these misconceptions, which can sink applications if applicants pander to them.

By way of example, imagine that you have worked in operations at a widget manufacturer. You have profound experience managing and motivating dozens of different types of people, at different levels, throughout your career, in both good economic times and bad. Even though your exposure to finance has been minimal, you erroneously determine that you need to be a “finance guy” to get into NYU Stern. So you tell your best, but nonetheless weak, finance stories, and now you are competing against elite finance candidates who have far more impressive stories in comparison. What if you had told your unique operations/management stories instead and stood out from the other applicants, rather than trying to compete in the school’s most overrepresented pool?

We think that attempting to defy business school stereotypes and truly being yourself—trying to stand out from all others and not be easily categorized—is only natural. Of course, for those of you who are still not convinced, allow us to share a quote from Stanford’s former director and assistant dean of MBA admissions, Derrick Bolton, who wrote on his admissions website, “Because we want to discover who you are, resist the urge to ‘package’ yourself in order to come across in a way you think Stanford wants. Such attempts simply blur our understanding of who you are and what you can accomplish. We want to hear your genuine voice throughout the essays that you write, and this is the time to think carefully about your values, your passions, your hopes and dreams.”

Makes sense, right?


mbaMission is the leader in MBA admissions consulting with a full-time and comprehensively trained staff of consultants, all with profound communications and MBA experience. mbaMission has helped thousands of candidates fulfill their dream of attending prominent MBA programs around the world. Take your first step toward a more successful MBA application experience with a free 30-minute consultation with one of mbaMission’s senior consultants. Sign up today atwww.mbamission.com/manhattangmat.

The post MBA Admissions Myths Destroyed: Business School Stereotypes appeared first on GMAT.
This Blog post was imported into the forum automatically. We hope you found it helpful. Please use the Kudos button if you did, or please PM/DM me if you found it disruptive and I will take care of it. -BB
Manhattan GMAT Online Marketing Associate
Joined: 14 Nov 2013
Posts: 272
Own Kudos [?]: 51 [0]
Given Kudos: 0
Send PM
How Many GMAT Problems Do I Need to Solve? [#permalink]
Expert Reply
FROM Manhattan GMAT Blog: How Many GMAT Problems Do I Need to Solve?


Did you know that you can attend the first session of any of our online or in-person GMAT courses absolutely free? We’re not kidding! Check out our upcoming courses here.

That’s a good question! Do you really need to solve all the GMAT problems in the Official Guide to the GMAT in order to score a 700? What about the other side of the issue: is it possible that there aren’t enough problems in the Official Guide? How many GMAT problems should you solve before taking the official GMAT?

Before I share my answer, let’s get some facts on the table.

First, there isn’t that much variety on the GMAT. I haven’t counted, but I’d bet that you need fewer than 50 math “rules” in order to solve almost every GMAT Quant problem. The number is probably similar for grammar rules and Sentence Correction. Likewise, on the Quant and Verbal sections, there are only five different types of problems. If you do every problem in the Official Guide, you’ll see just about every rule that you need for the GMAT, and you’ll see many examples of every problem type. All of the content on the test shows up in the Official Guide, even though it only contains a few hundred GMAT problems.

Despite that, the GMAT is a tough test. Why? Because, even with a very limited amount of content to draw from, the number of possible GMAT problems is infinite. Whether you’ve done a hundred practice problems or ten thousand practice problems, every problem you see on test day will be new to you. There’s no way around it—you need to get better at solving new problems from scratch.

That’s one advantage to doing a lot of GMAT problems. It lets you practice that exact skill: solving new problems that you’ve never seen before. It also lets you practice your timing, which is critical on the GMAT. There’s also something fun about spending a study session blasting through 20 or 30 problems: you walk away feeling like you’ve really gotten a lot of work done. However, there are disadvantages to studying by just doing a lot of GMAT problems. Here are a few of them.

First, it’s an inefficient way to study, especially when you’re first getting started. If you pick up the Official Guide and do 20 problems in order, there might be only one or two that really engage with your biggest weaknesses. There are various resources available that list the problems in the Official Guide by content area—try doing a set of 20 GMAT problems from a specific area you’re working on, instead. Or do some reading or drill sets from one of our GMAT Strategy Guides. (Not sure how to spot your GMAT weaknesses?  Check out this guide to analyzing a practice test.)

Focusing on getting a lot of problems done also discourages mastery. You don’t learn very much during the two minutes (or less) you spend doing a new problem. Learning happens afterwards, when you return to a problem thoughtfully and patiently. In fact, to learn the most you can from a tough problem, you should do it two or more times. Reviewing a problem in-depth takes a lot of time; if you commit to reviewing the GMAT problems you do, you won’t be able to do nearly as many problems in total. That’s okay. You’ll improve more in the limited time you have.

The biggest downside, though, is that making the same mistakes over and over will turn them into bad habits. It’s frustrating when you just keep making the same exact mistakes, time and time again. When you make a mistake, don’t just tell yourself that you’ll get it right next time and move on to the next problem set. Slow down. Analyze the problem. What do you need to change about your process? How could you avoid the mistake next time? If you make the same mistake more than once or twice, it might be time for some targeted sets of that particular type of problem. Pay attention to your mistakes, and work hard to change! You won’t improve by doing a lot of GMAT problems poorly.

Okay, so what exactly should you do? Here’s my advice. First, the Official Guides and the GMATPrep software (especially with the downloadable Question Pack) contain enough GMAT problems for almost anyone. You probably don’t need extra problems beyond that (although that would be a great thing to discuss with a GMAT tutor). Second, the way you study will change depending on how close you are to your test date. When you’re just getting started, if there’s content that you need to learn from scratch, focus on resources such as Foundations of Math and Foundations of Verbal, and do a lot of drill sets like the ones in those books. Drill sets aren’t like official GMAT problems—they’re similar, but they’re designed to test just one or two specific skills at a time.

As you keep studying, use practice tests to evaluate your weaknesses. Use the MPrep GMAT Strategy Guides, GMAT Interact, or even a 9-week GMAT Complete Course to address the areas you’re still struggling with. Incorporate some sets of random, timed GMAT problems, in order to practice timing and stamina. As you get closer to your test date, emphasize random problem sets more and more—but don’t forget to leave time to review problems. Doing a few well-chosen problems carefully, and really mastering them, will help you more than doing a hundred problems poorly.

Everybody’s heard about that one person who did five thousand GMAT problems and then scored a 780. But real success on the GMAT is both easier and harder than that. Unfortunately, there’s no magic number of problems that will guarantee you success. However, the number of problems you need to solve is probably smaller than you’d think! Focus on being thoughtful and thorough, rather than just “getting through” everything, and you’ll be on the right track.

Want more guidance from our GMAT gurus? You can attend the first session of any of our online or in-person GMAT courses absolutely free! We’re not kidding. Check out our upcoming courses here.

[b]Chelsey Cooley
is a Manhattan Prep instructor based in Seattle, Washington.
 [/b]Chelsey always followed her heart when it came to her education. Luckily, her heart led her straight to the perfect background for GMAT and GRE teaching: she has undergraduate degrees in mathematics and history, a master’s degree in linguistics, a 790 on the GMAT, and a perfect 170/170 on the GRE. Check out Chelsey’s upcoming GRE prep offerings here.

The post How Many GMAT Problems Do I Need to Solve? appeared first on GMAT.
This Blog post was imported into the forum automatically. We hope you found it helpful. Please use the Kudos button if you did, or please PM/DM me if you found it disruptive and I will take care of it. -BB
Manhattan GMAT Online Marketing Associate
Joined: 14 Nov 2013
Posts: 272
Own Kudos [?]: 51 [0]
Given Kudos: 0
Send PM
Columbia Business School Essay Analysis, 2017-2018 [#permalink]
Expert Reply
FROM Manhattan GMAT Blog: Columbia Business School Essay Analysis, 2017-2018

How can you write essays that grab the attention of MBA admissions committees? With this thorough analysis, our friends at
mbaMission help you conceptualize your essay ideas and understand how to execute, so that your experiences truly stand out.

For years, Columbia Business School (CBS) has been at the front of the pack of MBA programs that have been gradually shortening their application essays and requiring candidates to be direct and concise in their submissions. It was even the first school to incorporate a micro essay into its application. Last season, CBS gave applicants a bit more wiggle room with the essays, increasing the word count for each by a pretty significant margin (up to 100% in one case), but it has tightened the reins back down for this year’s applicants. Still, with a goal statement, three required essays, and an optional essay, you should have plenty of opportunities to convey a well-rounded impression of yourself for the admissions committee. Read on for our Columbia Business School essay analysis for the program’s 2017–2018 prompts…

Goal: What is your immediate post-MBA professional goal? (50 characters)
Including spaces, CBS’s career goal question is exactly 50 characters itself, showing candidates just how much (or how little, in this case) space they have to work with and just how succinct they are therefore expected to be with their response. At just 50 characters, this can hardly be considered a true essay, but you will need to approach this with the same level of thought and focus as any of your other written responses for the school. The prompt is clearly a no-nonsense request for information, with no room allowed for exposition, flowery words, background, or pandering. This is all about getting to the point and telling the admissions committee what it wants to know.

In the past, the school has provided a few sample responses, including “Work in business development for a media company” and “Join a strategy consulting firm,” illustrating that conveying the requested information in such a tight space is definitely doable and that you do not need to worry too much about grammatical issues (in other words, you do not need to start your statement with “I want to” or something similar). We like to offer the statement “Reveal true goals, not what you think Columbia Business School wants” as both our own example of keeping things concise and our advice on how to approach and fulfill this request. Think about what you truly want to do with your career in the short term and state this aspiration directly. Keep in mind that the rest of your application will need to provide evidence that your stated goal aligns with your existing skills and profound interests, especially once they have been augmented by an MBA education. This will show that your professed goal is achievable and lend credibility to your statement. If you can do this in 50 characters (not words!), you will have done what you need to answer the school’s question quite well.

Essay #1: Through your resume and recommendations, we have a clear sense of your professional path to date. What are your career goals over the next 3 – 5 years and what, in your imagination, would be your long-term dream job? (500 words)
CBS starts this essay question by more or less telling you not to recap your career to date, so we strongly recommend that you do so (and briefly, at that) only if context is absolutely needed for your stated goals to be understood and/or believable—perhaps if you are making a fairly remarkable career change. Pay particular attention to the phrases “dream job” and “in your imagination” with respect to the long-term portion of the question. The school is prompting you to be creative and perhaps even to challenge or push yourself to think big. Columbia Business School wants individuals who do not just follow prescribed paths according to someone else’s blueprint but who are aspirational and more inclined to forge their own way. This is not to suggest that if you have a more traditional plan in mind that you are in trouble or at risk of losing the admissions committee’s attention, but you may need to take a little extra time to consider your ambitions from the perspective of “what if?” and delve more deeply into what you hope to achieve to find the more personal and inspiring elements of your goals. Showing creativity and individualism here can only be helpful.

Although this is not a request for a textbook personal statement essay, your response will certainly involve some elements of the topics covered in such a submission, such as short- and long-term goals. The mbaMission Personal Statement Guide offers advice on brainstorming and crafting such essays along with multiple illustrative examples and so may be helpful in preparing your CBS response to this prompt. You can download your free copy here.

Columbia Business School does not explicitly ask how its MBA program will factor into the achievement of your goals, but if you feel that particular resources the school offers could or will be uniquely influential and advantageous to you as you advance along your path, we believe you have sufficient room and leeway to mention these. However, generic claims or empty pandering have no place at all in this rather compact essay. Any CBS resources you reference must be specific to your needs, and the cause-and-effect relationship between these resources and your anticipated success must be very clear. For example, an applicant might discuss the appeal and instrumentality of CBS’s Value Investing Program and 5x5x5 Student Portfolio Fund in his or her aspirations to one day break into the asset management world or later launch a hedge fund. We do not recommend going so far as to dedicate an entire paragraph to discussing school resources, but you might consider thoughtfully embedding a relevant reference or two into your submission to acknowledge the program’s role in achieving your stated career intentions. Or should we say dreams?

Essay #2: The full-time MBA experience includes academics, recruiting and networking. What are your personal priorities and how do you anticipate allocating your time at Columbia Business School? (250 words)
Start your brainstorming for this essay by first considering your priorities within the three areas CBS specifically notes—academics, recruiting, and networking—but do not feel compelled to limit yourself to them if you have other ideas or plans. The school wants well-rounded students who will not “silo” themselves into just one area and who anticipate using multiple aspects of the MBA program to their and others’ advantage. If you care only about the academic aspect of business school and do not envision yourself participating in any club activities or availing yourself of events or resources outside the classroom, you might not be the kind of candidate top-tier schools such as CBS are seeking. Earning your MBA involves more than completing business classes, and the admissions committee strives to construct a diverse class of engaged and experienced people who can learn both together and from one another. This requires that everyone participate and contribute, and not just in class.

With your response to this essay prompt, show Columbia Business School where and how you expect to be active in its community and program, whether that is via a certain club, event, course, or other avenue. Explain what drives you toward these areas and activities and what you imagine your involvement will look like. If you can frame your vision in a way that reveals a benefit for those around you as well, this is even better. For example, will you commit a large portion of your time to your job search because landing a highly coveted role at an elite Wall Street firm is your main impetus for getting your MBA? And if so, will you offer to run mock interviews with your fellow students who share this dream? Perhaps you are interested in joining the CBS Follies group to fulfill your artistic and dramatic side and balance your quant-heavy course work, and your many years in the theater will help you coach classmates who are new to the stage so that they can fully benefit from and enjoy the experience as well.

Keep in mind that the specific activities and areas you choose are not what is important here. CBS is not choosing people based on whether they expect to populate certain clubs or organize student conferences, so the admissions committee will neither ding nor reward you for choosing one option over another. What is important is that you show you have a true understanding of what CBS offers and a plan of attack for your experience within the program. After reading your essay, an admissions officer should feel that you really “get” CBS and can clearly envision yourself there. The school knows that an applicant who has dedicated the time and effort necessary to develop advanced knowledge of the program is one who will have a successful experience. In giving you a place in the class, the admissions committee is essentially betting that you will thrive more at CBS than thousands of other candidates would, so show that you are a low-risk, high-reward potential admit.

EMBA Essay #2: Columbia Business School’s Executive MBA will challenge you by offering a rigorous academic experience, global exposure through the international seminar, and the opportunity to immediately apply what you learn to your career. How will you approach balancing the demands of the program with your professional and personal life while you are in school? (250 words)
This question basically exists because CBS wants to be sure you truly intend to finish the program. We cannot claim to have specific numbers on this phenomenon, but we know that EMBA students sometimes get overwhelmed by the demands of balancing work, studies, and personal/family life. As a result, a number of them ultimately drop out of the program each year, including some who are asked to leave the program by their employers, who also did not understand the time demands involved. With this prompt, CBS is essentially asking, “Are you sure you know what you are getting into?”

To help assuage the admissions committee’s fears on this point, you might take a somewhat procedural approach in your response, explaining that you anticipate dedicating certain hours on certain days, with your manager’s approval, to study and complete your course work for the program. You might address how you have engaged supportive stakeholders—such as partners, children, and friends—in discussions about your commitment to help clarify and calibrate expectations. Having read your response, the admissions reader should feel comfortable that you have a clear plan in mind, that this is not a whimsical choice but a well-informed decision that oozes commitment. Columbia Business School wants to know that you will see the program through to the end, and to communicate this effectively, you must demonstrate that you have the process and support mechanisms in place to not only finish it but thrive within it.

Essay #3: Please select and answer one of the following essay questions: (250 words)
CBS has replaced its “What will your Clustermates be pleasantly surprised to learn about you?” query from last season with this new set of prompt choices, both of which are still invitations to share aspects of your personality but are notably broader in scope. Whichever one you ultimately select, focus on giving the school insight into what makes you tick as an individual, beyond your professional skill-set and goals.

a: Please tell us what you feel most passionate about in life.
Although we acknowledge that you might actually be “most passionate” about your career, this is not your best choice for a topic here, especially given that two of the other essays have already allowed you to discuss your professional life. Clearly, if you are applying to and ready to complete a leading MBA program—challenges not for the faint of heart—you are a driven individual with more enthusiasm and ambition career-wise than the average person. The Columbia Business School admissions committee already knows this. What it wants to learn now is what gets your heart pumping and mind racing outside of work. As Steve Jobs once said, “People with passion can change the world,” and although he was speaking about careers at the time, the statement is true for all aspects of one’s life. Passion is inspiring and energizing and can lead to big ideas and actions. Sharing with the school where your passion lies gives the admissions committee an idea of where you might someday make an impression on the world, how you might leave your mark—especially once you are equipped with all you will gain and learn during your MBA experience.

That said, do not worry if your passions seem commonplace. For example, perhaps you feel passionate about basketball. Because this is an experience that anyone could share and enjoy, it might seem pedestrian. The key, however, is not what inspires you but how you engage with it. If you can show that basketball is not just a hobby you simply enjoy from time to time but is instead something you connect with on a deep level and in various ways—perhaps having played for many years, you now coach youth teams in your community and have amassed a truly impressive trading card or jersey collection—then this initially unremarkable-seeming choice most definitely becomes an acceptable discussion topic. Think about your possible choices in terms of intensity, enthusiasm, devotion, longevity, loyalty, excitement, and heart, and be honest with yourself. The elements of your life that inspire and align with these concepts could be appropriate fodder for this essay, while anything that does not should be immediately discarded as an option.

Once you have identified the passion you wish to discuss, avoid simply telling the admissions committee about it and instead demonstrate how this passion manifests in your life. For example, rather than stating, “I have been watching and playing basketball since I was a child,” you need to create a more vivid impression of your dedication and involvement, such as “From playing with my brothers after school to varsity ball in college to now coaching a youth league in my community, I can hardly remember a time when basketball wasn’t an integral part of my life.”     

b: If you were given a free day and could spend it anywhere, in any way you choose, what would you do?
Like option A, this prompt asks for a window into your personal self, but with a bit of a relaxed twist. It conveys slightly less of an idea of singularly focused drive and intensity and more one of a variety of activities and experiences that would generate inner happiness and satisfaction. Whether you want to spend your free day jumping out of a plane, swinging a hammer building homes for Habitat for Humanity, or reading quietly beside a lake somewhere, the activity you share is not the admissions committee’s focus so much as what your choice says about who you are and how you like to live your life. In this essay, you want your interests to reveal that you are an appealing and likeable character—one who spends his or her time thoughtfully and productively, rather than being easily bored, indecisive, or in need of an outside source to provide distraction. Help CBS understand your internal motivations and values and how they may come into play both in the program and beyond your two years as an MBA student. And as with option A, you will want to take a “show, rather than tell” approach in your writing. Avoid simply stating your anticipated agenda outright, and instead strive to really bring the admissions reader along with you on this imaginary day off via your descriptions.

Optional Essay: Is there any further information that you wish to provide the Admissions Committee? If so, please use this space to provide an explanation of any areas of concern in your academic record or your personal history. You may submit bullet points. (Maximum 500 words)
This optional essay question starts out sounding like an open invitation to discuss almost anything you feel like sharing with the admissions committee, but the second line (which was not part of the prompt last season) dials things in and puts the spotlight on addressing problem areas specifically. The additional directive about bullet points seems to be a not-too-veiled implication that the school wants you to focus on imparting key information rather than offering a detailed and long-winded explanation of the issue in question. Without a doubt, this is not an opportunity to share another cool story or otherwise try to impress or pander to the admissions committee. If you do not truly need to explain an issue or potentially confusing element of your candidacy (a poor grade or overall GPA, a low GMAT score, a gap in your work experience, etc.), we do not recommend that you submit an optional essay; if you do have issues to clarify, keep things concise. In our mbaMission Optional Essays Guide, we offer detailed advice on when and how to take advantage of the optional essay, with multiple examples, to help you mitigate any problem areas in your profile.

mbaMission is the leader in MBA admissions consulting with a full-time and comprehensively trained staff of consultants, all with profound communications and MBA experience. mbaMission has helped thousands of candidates fulfill their dream of attending prominent MBA programs around the world. Take your first step toward a more successful MBA application experience with a free 30-minute consultation with one of mbaMission’s senior consultants. Sign up today at www.mbamission.com/manhattangmat.

The post Columbia Business School Essay Analysis, 2017-2018 appeared first on GMAT.
This Blog post was imported into the forum automatically. We hope you found it helpful. Please use the Kudos button if you did, or please PM/DM me if you found it disruptive and I will take care of it. -BB
Manhattan GMAT Online Marketing Associate
Joined: 14 Nov 2013
Posts: 272
Own Kudos [?]: 51 [0]
Given Kudos: 0
Send PM
Past Participles on GMAT Sentence Correction [#permalink]
Expert Reply
FROM Manhattan GMAT Blog: Past Participles on GMAT Sentence Correction


Did you know that you can attend the first session of any of our online or in-person GMAT courses absolutely free? We’re not kidding! Check out our upcoming courses here.

Check out these two sentences:

The horse raced past the barn.

The horse raced past the barn fell.

Believe it or not, both sentences have good grammar. But one of them makes a lot more sense than the other one! Let’s break them down and understand why.

In the first sentence, the subject is “the horse.” Then, there’s a verb in the past tense: “raced.” The core of this sentence—the part of the sentence that tells you who did what—is “the horse raced.” There’s also a modifier, which tells you where the horse raced.

In the second sentence, it looks like you’ve got the same subject and the same verb. However, once you get to the end of the sentence, there’s another verb, “fell.” A sentence can’t have two main verbs, not without a connector, such as “and,” in between them. There must be something strange going on. And in fact, there is.

In this sentence, the phrase “raced past the barn” isn’t part of the core. Instead, it’s a modifier—a very tricky modifier. To see how that works, let’s rewrite the sentence slightly:

The horse, which was raced past the barn, fell.

Now, it should be clearer that the “raced past the barn” part of the sentence is a modifier, and the core is actually “the horse fell.” In other words, the sentence says that a horse fell—why? Because a jockey (or someone) raced it past the barn.

In the tricky sentence, the word “raced” is a past participle. Past participles are tough for your brain. That’s because in English, they usually look just like normal past tense verbs. Since normal verbs are much more common than participles, your brain isn’t expecting to see a participle when you start reading. When you see the first three words of the sentence—“the horse raced”—you naturally assume that you’ve just read the main subject and verb of the sentence. Then, when you find a second verb, you get confused. You have to go back and “re-understand” the first part of the sentence. This effect is called “garden pathing.”

Here’s a summary so far:

  • Past participles are verbs that usually end in “-ed.”
  • A past participle can start a modifier.
  • Past participles are tricky, because they look the same as regular past tense verbs. If you assume that a past participle is actually a regular verb, you’ll read the sentence incorrectly at first.
Here are some examples of good sentences where an “-ed” verb is the first word of a modifier. In every sentence, the modifier appears in bold.

  • The whale named Willy was finally free.
  • Sneha, accompanied by her sister Jaina, traveled to Milwaukee.
  • The student awarded first place was overcome by pride.
  • Several competitors selected for the finals were unable to attend.
Some of these sentences were probably easier for you to understand than others. However, they all have something in common. They all have an “-ed” verb right next to a noun, and then they have another past tense verb, later in the sentence. Let’s dissect sentence number 3.

The student awarded first place was overcome by pride.

You might start reading this one and immediately assume that the student gave someone an award. That reading makes sense until you hit the word “was.” Then, the sentence seems strange. If the student was the one giving the award, shouldn’t the sentence end at this point? Let’s reevaluate the sentence. Awarded first place is actually a modifier—an “-ed” modifier—that describes the student. The core of the sentence is as follows:

The student was overcome.

Think you’ve got it? Open up your 2017 Official Guide to the GMAT  and try the following Sentence Correction problems.

Sentence Correction # 680, 695, 716, 733, 743, 746

Some of these are easier than others, but they all include past participles, and I’ve seen students get tricked by all of them. If you get most of them right, you’re doing great!

Want more guidance from our GMAT gurus? You can attend the first session of any of our online or in-person GMAT courses absolutely free! We’re not kidding. Check out our upcoming courses here.

[b]Chelsey Cooley
is a Manhattan Prep instructor based in Seattle, Washington.
 [/b]Chelsey always followed her heart when it came to her education. Luckily, her heart led her straight to the perfect background for GMAT and GRE teaching: she has undergraduate degrees in mathematics and history, a master’s degree in linguistics, a 790 on the GMAT, and a perfect 170/170 on the GRE. Check out Chelsey’s upcoming GRE prep offerings here.

The post Past Participles on GMAT Sentence Correction appeared first on GMAT.
This Blog post was imported into the forum automatically. We hope you found it helpful. Please use the Kudos button if you did, or please PM/DM me if you found it disruptive and I will take care of it. -BB
Manhattan GMAT Online Marketing Associate
Joined: 14 Nov 2013
Posts: 272
Own Kudos [?]: 51 [0]
Given Kudos: 0
Send PM
The GMAT Official Guide 2018 Edition, Part 4 [#permalink]
Expert Reply
FROM Manhattan GMAT Blog: The GMAT Official Guide 2018 Edition, Part 4


Did you know that you can attend the first session of any of our online or in-person GMAT courses absolutely free? We’re not kidding! Check out our upcoming courses here.

In the previous three installments of this series, I summarized some of the big messages and discussed some of the interesting problems I spotted in the GMAT Official Guide 2018. (If you’d like, you can start with the first installment and work your way back here.)

Today, I’ve got lists for you—the problems that are new to the GMAT Official Guide 2018 (by chapter and problem number).

Question Difficulties
As always in the OG, the question number indicates the relative difficulty level—higher-numbered questions within one chapter are generally harder than lower-numbered questions in that same chapter. If you compare the question numbers of the dropped vs. new questions in the two Quant chapters, you may notice that a greater number of higher-level questions were dropped than were replaced—in other words, the average difficulty of Quant questions in the GMAT Official Guide 2018 has decreased a bit. (Of course, there are still plenty of hard questions in the book.)

We’ve heard that there are plans to release a future product with a focus on higher-level / harder Quant questions. I don’t have an actual release date for you, so until that product is released, I would first direct you toward the official GMAT Focus product for even harder Quant questions—with one caveat.

GMAT Focus is adaptive; in other words, you get what you earn. If your Quant skills really are up there, then great—you’ll earn the harder (or hardest!) questions in GMAT Focus. If your Quant skills aren’t there yet but you just feel like you “should” study harder questions, then suppress that impulse for now. First, do what you need to get your skills up before you dive into GMAT Focus. (I would argue that, in this circumstance, you might “feel like” you should do harder stuff, but you’re not actually ready for it yet, since you can’t yet earn that material in an adaptive setting. There are still things to be learned at a lower level, so don’t try to shortcut the overall learning process.)

As I mentioned in the first part of this series, the Diagnostic chapter (chapter 3) has not changed at all. The five question-type-specific chapters have all added new questions. Here they are!

Problem Solving (Chapter 5)


Data Sufficiency (Chapter 6)


Reading Comprehension (Chapter 7)
Note: RC data is grouped by passage.



Critical Reasoning (Chapter 8)


Sentence Correction (Chapter 9)


That’s it—all 122 new questions in the GMAT Official Guide 2018. Have fun trying them out. Go forth and learn! 

Can’t get enough of Stacey’s GMAT mastery? Attend the first session of one of her upcoming GMAT courses absolutely free, no strings attached. Seriously.


Stacey Koprince is a Manhattan Prep instructor based in Montreal, Canada and Los Angeles, California. Stacey has been teaching the GMAT, GRE, and LSAT  for more than 15 years and is one of the most well-known instructors in the industry. Stacey loves to teach and is absolutely fascinated by standardized tests. Check out Stacey’s upcoming GMAT courses here.

The post The GMAT Official Guide 2018 Edition, Part 4 appeared first on GMAT.
This Blog post was imported into the forum automatically. We hope you found it helpful. Please use the Kudos button if you did, or please PM/DM me if you found it disruptive and I will take care of it. -BB
Manhattan GMAT Online Marketing Associate
Joined: 14 Nov 2013
Posts: 272
Own Kudos [?]: 51 [0]
Given Kudos: 0
Send PM
Mission Admission: Use Your Judgment on MBA Application Essay Details [#permalink]
Expert Reply
FROM Manhattan GMAT Blog: Mission Admission: Use Your Judgment on MBA Application Essay Details


Mission Admission is a series of MBA admissions tips from our exclusive admissions consulting partner, mbaMission.

“Should I use Calibri or Times New Roman font for my essays?”

“Should I list my GPA to the third or fourth decimal place?”

“I don’t have enough space to enter my full title, so should I write ‘Vice President’ or ‘VP Sales’?”

As candidates approach the MBA application essay season, small questions start to arise—questions that often require using judgment to answer. We can safely say that no one was ever kept out of Harvard Business School for listing his/her GPA to the third decimal point or for abbreviating a title. Remember, the admissions officers are not punitive. They are not mean-spirited people, reading your application and searching for reasons to reject you. So, if you have a small lingering question about your application, you can connect with the Admissions Office and ask someone there. Most often, they will ask you to use your judgment. As long as your broad story is compelling, the smallest details should take care of themselves.


mbaMission is the leader in MBA admissions consulting with a full-time and comprehensively trained staff of consultants, all with profound communications and MBA experience. mbaMission has helped thousands of candidates fulfill their dream of attending prominent MBA programs around the world. Take your first step toward a more successful MBA application experience with a free 30-minute consultation with one of mbaMission’s senior consultants. Sign up today atwww.mbamission.com/manhattangmat.

The post Mission Admission: Use Your Judgment on MBA Application Essay Details appeared first on GMAT.
This Blog post was imported into the forum automatically. We hope you found it helpful. Please use the Kudos button if you did, or please PM/DM me if you found it disruptive and I will take care of it. -BB
Manhattan GMAT Online Marketing Associate
Joined: 14 Nov 2013
Posts: 272
Own Kudos [?]: 51 [0]
Given Kudos: 0
Send PM
Know the GMAT Code: Translation Traps [#permalink]
Expert Reply
FROM Manhattan GMAT Blog: Know the GMAT Code: Translation Traps


Did you know that you can attend the first session of any of our online or in-person GMAT courses absolutely free? We’re not kidding! Check out our upcoming courses here.

The problem we’re going to talk about today is a work of art. (Yes, I’m a geek. Did you not know that already? )

But I’m serious: it’s a thing of beauty. It looks super easy. It’s not—there are traps all over the place. The GMAT test writers have a genius for tying us into knots!

Try out this next problem in our Know the GMAT Code series and then we’ll talk about the awesome lessons here. Note: this one’s from the GMATPrep® free exams.

“*Of the 60 animals on a certain farm, ²/³ are either pigs or cows. How many of the animals are cows?

“(1) The farm has more than twice as many cows as it has pigs.

“(2) The farm has more than 12 pigs.”

(If you haven’t done Data Sufficiency before or are new enough to DS that you’re wondering where the answer choices are, start here and come back to this article later.)

Let’s do this!



1-second Glance. DS. Story. Will need to translate.

Read. The story seems pretty straightforward. I feel like this is a question they’d give me in 5th grade…I better be careful!

Let’s jot this stuff down and see what’s going on.



Reflect…Okay, so only two-thirds of the animals are pigs or cows. What about the rest? I don’t know—I guess I just know that there are other animals that aren’t pigs or cows. I’ll table that for now, but I’ve got it in the back of my mind in case it comes up later with the statements.

Oh, and I can calculate one thing: two-thirds of 60 is 40, so there are 40 total animals that are either pigs or cows.

Statement (1) is a little confusing. I think I’m going to start with statement (2).

“(2) The farm has more than 12 pigs.”



Time to test some cases. If there are 13 pigs…



At whatever point you can tell that it’s possible to get more than one value for the number of cows, you can stop. Cross off the top row (BD) and move on to the other statement.

Note: That middle column, v?, stands for “is this a valid case to test?” You are only allowed to try numbers that make the statement that you’re testing true. I know it seems a little silly to make a separate column for that on this problem, but if I’ve learned one thing over the years, it’s this: careless mistakes are the bane of any standardized test-taker’s existence. Have a process. Follow the process—every time. Trust the process.

Okay, so answers (B) and (D) are out; time to test statement (1).

“(1) The farm has more than twice as many cows as it has pigs.”

Er. How does that get translated? If there are exactly twice as many cows as pigs, then the equation would be c = 2p. But that’s not what it says. Rather, there are more than twice as many. Is that just an inequality?

Try it out. If the equation is c > 2p, do real-life, logical numbers work? If you have 4 pigs, you’d have to have 9 or more cows. 9 > 2(4) is true. If you have 4 pigs, you couldn’t have just 8 cows. 8 > 2(4) is false.

Okay, this is the right equation (or, technically, inequality): c > 2p. If you’re ever not sure, take your best guess on the translation and then test it with some real numbers to see whether it makes sense.

So, is this statement enough? Test it out!



I said earlier that you’re only allowed to test numbers that make the statement that you’re testing true. But there’s actually one more thing you have to do. If the question stem gives you any true (or what’s called “given”) information, you have to make that true, too.

In my first case for statement (1), p and c don’t add up to 40. So let me try that again.



Okay, same deal as statement (2). There are at least two possible values for the number of cows, so this statement isn’t sufficient, either. Cross off (A).

Time to try the two statements together.

Hmm. First, according to statement (2), p has to be at least 13. So, according to statement (1), c has to be at least 13(2) + 1 = 27.

Oh, and those two numbers do add up to the right total: 13 + 27 = 40. Great! So this is one possible set of values for p and c. Is there another?

If p is 14, then c has to be at least 14(2) + 1 = 29. So there is a second value…wait! 14 + 29 = 43. That’s the wrong total. And as I keep increasing p, I’m going to keep increasing the total, so there’s no other pair of numbers that will properly add up to 40.

This is it: c has to be 27. The two statements together are sufficient.

The correct answer is (C).

That wasn’t crazy math or anything. Why did I say this question is harder than it looks?

It has to do with the traps. First of all, they tell us there are 60 animals but then go on to say that ²/³ are either pigs or cows…in other words, the real total number is only 40, not 60. If you work with the number 60, you’re going to think that more than one pair of numbers is possible and get (E) as your answer.

Even if you’re fully up to speed on the 40, you have to remember to bring that fact back in as a “check” at the end. Otherwise, you’re going to think: could be 13 and anything over 27, so there are multiple possible values for c…and you’re going to get (E) again.

It’s also super easy to roll right over the “more than” in statement (1). The test has been going on for a couple of hours now, you’re starting to get mentally fatigued, you might be worried about timing and thinking, “Yes, this is an easy one! I can save time!” That’s where careless mistakes pounce.

If you think statement (1) just says twice as many, not more than twice as many, you’ll think the answer is (A). The only thing that might help you catch that mistake is if you try to do the math: if you have 40 cows and pigs, then you’d have to have…13 pigs and 26 cows? That doesn’t add up right. There’s no way to get to 40 using whole numbers of pigs and cows. But you may not even try to do that math because, after all, this is Data Sufficiency! And if you think the total is 60, then the math does add up: 20 pigs and 40 cows, still leaving you with incorrect (A).

Key Takeaways for Knowing the GMAT Code
(1) Easier-looking does not necessarily mean easier. A lot of times, easier-looking problems have some easy-to-fall-for traps. Work carefully—don’t get ahead of yourself. Write everything down. Map out the problem.

(2) Turn any knowledge you gain into Know the GMAT Code flash cards:



*GMATPrep® questions courtesy of the Graduate Management Admissions Council. Usage of this question does not imply endorsement by GMAC.

Can’t get enough of Stacey’s GMAT mastery? Attend the first session of one of her upcoming GMAT courses absolutely free, no strings attached. Seriously.


Stacey Koprince is a Manhattan Prep instructor based in Montreal, Canada and Los Angeles, California. Stacey has been teaching the GMAT, GRE, and LSAT  for more than 15 years and is one of the most well-known instructors in the industry. Stacey loves to teach and is absolutely fascinated by standardized tests. Check out Stacey’s upcoming GMAT courses here.

The post Know the GMAT Code: Translation Traps appeared first on GMAT.
This Blog post was imported into the forum automatically. We hope you found it helpful. Please use the Kudos button if you did, or please PM/DM me if you found it disruptive and I will take care of it. -BB
Manhattan GMAT Online Marketing Associate
Joined: 14 Nov 2013
Posts: 272
Own Kudos [?]: 51 [0]
Given Kudos: 0
Send PM
Tiny GMAT Critical Reasoning Mistakes You Might be Making (Part 2) [#permalink]
Expert Reply
FROM Manhattan GMAT Blog: Tiny GMAT Critical Reasoning Mistakes You Might be Making (Part 2)


Guess what? You can attend the first session of any of our online or in-person GMAT courses absolutely free—we’re not kidding! Check out our upcoming courses here.

As Hemingway did for lions on the Savannah, I have continued my hunt in GMAT Critical Reasoning problems for little mistakes in logic, the tiny tempting answer choices that could trap even the most rational of minds. I have also been consuming as much whiskey as he would have, so plese forgive any typps cos Im perty drnk rite now…

If you missed our first post on the subject, or if you need a brush up, read it here to learn three ways you might slip up on GMAT Critical Reasoning.

Since then, I have found more. As I did last time, I’ll lay out an example that contains the trap, explain the logic behind it, and point you to an actual CR problem that has the same kind of issue. Try to find the right answer to that question and the trap answer that fits the given description.

How Might an Answer Choice Trap You on GMAT Critical Reasoning?
1) It Shifts to or from a Subgroup
Companies have not sold video-game systems that incorporate 3D technology at nearly the levels that they had predicted. Many companies have decided that they should stop investing in hardware that allows 3D gameplay. However, Sow-Knee, a company that makes both televisions and video game systems, is developing a TV screen that has 3D imaging that does not require users to wear glasses, something that has irritated users in the past. Therefore, the head of Sow-Knee’s gaming division has decided to continue developing games for 3D display.

Which of the following would be most useful in evaluating the argument?

A) Do people who play video games like to diversify which brands they buy their home electronics from?

B) Have some people never purchased video game systems because playing video games makes them nauseous and they end up vomiting all over their couches?

You might think that if video games make people projectile vomit then Sow-Knee should stop investing in 3D gaming. However, we were never discussing the whole market of video game systems—we were talking about video game systems that incorporate 3D technology. Similarly, we were never worried about people who can’t play video games at all. Answer choice B has shifted away from that scope. You might think “Oh man, 3D, nausea, this is something we gotta consider!” But those people are already avoiding all video games, and we’re talking about a subgroup of video games and the people who buy video game systems.

This is one of those times where a general explanation is, “That’s out of scope,” but you will want to try to specify why it’s out of scope. In this particular case, the scope was a subgroup. We probably don’t want to switch to another subgroup, and we want to be careful when we move from our subgroup to the group-at-large, because that might take us out of the scope of our argument.

A good indicator that something like this might be going on are words like ‘who,’ ‘that,’ ‘which,’ ‘whose,’ or any other word like this that will specify which ‘type’ of a specific ‘thing’ you’re discussing: nuns who use chainsaws, watches that have rocket launchers—these designate that you’re talking about a specific type of thing.

(EXAMPLE: CR 642 about drinking wine)

2) It Gives Information in a Logically Negated Form
Some homes in the area that have structural issues have been found to be infested with termites. Termites are wood-eating insects that reproduce quickly and can cause load-bearing beams to hollow out, seriously increasing the risk of a collapse. To prevent such collapses from happening, the homeowners association is advising local homeowners to have exterminators visit their house twice a year to spray for termites.

Which of the following, if true, casts the most doubt that the homeowners association’s plan will have the intended effect?

A) Many homeowners will follow the city government’s advisements.

B) Several houses with structural issues—including many that had roof collapses—live in areas where termite infestation rates are very low.

“If the answer’s not B, I’ll eat my foot.”

…How would you like it prepared?

“SON OF A—“

NO, THINK OF THE CHILDREN!

I get it. This one is especially mean (probably meaner than the GMAT). B has just got to weaken the argument, right? The association is saying to spray for termites, and B tells me a lot of the houses that have fallen haven’t even had termites!

But notice that the argument is about collapses that have been caused by termites. The fact that other collapses haven’t been caused by termites is a shift of subgroups talked about above.

“I hate you.”

Well established, and appreciated.

“Okay, but what about A? How does that weaken the argument, if people are going to follow the government’s plan?”

Well, let’s look at A very closely. What is the logical opposite of “Many homeowners will follow the plan”?

Here’s a hint: many homeowners is a far cry from all homeowners.

The logical opposite is: “Some homeowners will not follow the plan.” If some homeowners do not follow the plan, then it makes it less likely that these termite-caused cave-ins will be prevented.

Also realize that in answer choice B, several collapses occurring in low-termite-infested areas means that some other collapses occurred in high-termite-infested areas.

This is very tough. One way to spot this trap is to look out for words like ‘no,’ ‘none,’ ’few,’ ‘some,’ ‘several,’ ‘many,’ ‘most,’ and ‘all.’

In general, consider what negating the answer does to the argument. Sometimes, the negated form of the sentence is obviously correct, so they hide it with this little logical twist.

(And just to drive the point home: the argument is about prevention, not reduction. This is a binary argument).

(EXAMPLE: CR 643, about business loans)

3) It Strengthens the Argument Instead of Stating an Assumption
Literary critics label certain books ‘guilty pleasures.’ What they mean by this is that a book might not have great literary merit, but is enjoyed because it is low-brow, perhaps even pornographic. However, this is a false distinction and a subtle attempt to diminish the value of these books, which are just as valuable as those that these critics call ‘high literature.’ Writing these ‘guilty pleasures’ takes as much time and talent as writing the pretentious novels these snobbish critics adore so much.

Which of the following is an assumption on which the argument depends?

A) A majority of people say their favorite novel is one that literary critics call a ‘guilty pleasure.’

B) The value of an achievement is derived at least in part from the duration of its creation.

This particular mistake could be generalized as, “It’s the right answer to the wrong problem.” We’ve all been there—it’s a weaken question, but somewhere in working through the twisted logic we forgot that and picked an answer that strengthens. However, this particular version of that error is the one I consider most tempting.

This is an assumption question. We must remind ourselves what a GMAT assumption is and what role it plays. The assumption is an unstated premise to the argument that actually holds the entire argument together. It’s not something that makes the argument better or stronger or more likely; it is something that lets the argument function at all. If the assumption weren’t true, the argument would pretty much be ruined. So a particularly tricky wrong answer choice is an answer that makes my argument better, but that isn’t absolutely necessary to it.

Answer choice A strengthens my conclusion that this ‘cheap literature’ does, in fact, have a lot of value. But my argument could still hold even if every single guilty pleasure novel got lost among the tattering, decaying pages of Twilight and The Firm. I need an assumption that, if false, ruins my argument. Well, if the value of an achievement is not derived from how long it took to make, then my argument falls apart—I was saying these works have value because they take just as long to write. If that’s not where value comes from, then my argument is totally moot, and the fact that people like these novels doesn’t really matter.

(EXAMPLE: CR 607, about birdlike dinosaurs)

So there you go. Three more tempting answers to keep your eyes out for. I’m quite sure I’ll find more for a part 3. If you think you’ve found something that fits the bill, let us know! Until then, enjoy your foot medium rare with a nice, strong whiskey.

EXPLANATIONS:
CR 642: A and D are both pretty tempting wrong answers, but note that this argument is talking specifically about sulfite allergies and the people who have them. We just don’t care about other substances that might cause an allergic reaction.

CR 643: C is a very, very tempting wrong answer (it’s the one I picked when I first did this problem). It’s actually chosen more often than the right answer, something that is very rare and indicates a real doozy of a question. But if some people choose not to increase their levels of saving, that means some others will increase their levels of savings, so there will be more money saved to loan out to businesses. Also, answer choice D is incorrect for one of the reasons mentioned in this post. Which one do you think it is?

CR 607: Answer choice A not only strengthens the argument—it guarantees it. If there are no descendants of the birdlike dinosaurs, then birds definitely aren’t descendants of the birdlike dinosaurs. But this answer is not something the argument totally depends on. That is, it’s possible that the birdlike dinosaur does have a descendant (a birdlike hippo, for example), but the argument that birds didn’t evolve from birdlike dinosaurs could still hold.

Want some more GMAT tips from Reed? Attend the first session of one of his upcoming GMAT courses absolutely free, no strings attached. Seriously.

Reed Arnold
 is a Manhattan Prep instructor based in New York, NY.
 He has a B.A. in economics, philosophy, and mathematics and an M.S. in commerce, both from the University of Virginia. He enjoys writing, acting, Chipotle burritos, and teaching the GMAT. Check out Reed’s upcoming GMAT courses here.

The post Tiny GMAT Critical Reasoning Mistakes You Might be Making (Part 2) appeared first on GMAT.
This Blog post was imported into the forum automatically. We hope you found it helpful. Please use the Kudos button if you did, or please PM/DM me if you found it disruptive and I will take care of it. -BB
Manhattan GMAT Online Marketing Associate
Joined: 14 Nov 2013
Posts: 272
Own Kudos [?]: 51 [0]
Given Kudos: 0
Send PM
How to Study for the Executive Assessment (Part 3) [#permalink]
Expert Reply
FROM Manhattan GMAT Blog: How to Study for the Executive Assessment (Part 3)


Did you know that we offer an Executive Assessment Masterclass? In one weekend, we’ll give you the tools you’ll need to ace the EA and take your career to the next level.

Welcome to the third and final installment of our series on the Executive Assessment! If you’re just joining us, start with Part 1 and work your way back here.

So far, we’ve talked about what’s on the Executive Assessment, how it’s scored, and how the adaptive nature of the exam works. It’s time to talk about time management, including how to manage the fact that we can answer questions within one panel in whatever order we please.

Time Management Basics on the Executive Assessment
Each section is 30 minutes long and has either 12 or 14 questions, so you have a bit over 2 minutes, on average, to answer each question.

As you work through any section, you’ll be given the option to mark questions. You can see a list of the marked questions and then click to jump right back to a particular question. This is a great feature as long as you know when and how to use it—and when and how not to use it.

First, when you do mark something, also put in a random answer right now. After all, you might not make it back later.

Next, you’re not going to know how to do everything, so you will want to guess on some questions as you go—and never return to these questions. Any time management strategy includes some “bail” categories: If you see certain things that you know are big weaknesses, you’re going to guess immediately, or bail—and that will give you a little more time to spread across the questions that you are going to try to get right.

We also have to account for the fact that IR is given all at once, while Verbal and Quant are given in sets of 2 panels each. Let’s see how this all plays out section by section.

Time Management: Integrated Reasoning
In IR, bailing on 2 or 3 questions will leave you just 9 or 10 questions to do in 30 minutes. Further, you don’t have to worry about having two separate panels in this section, so just start working through in order, while looking for opportunities to mark questions or to bail on questions.

Let’s start with “bail” questions. “Bail” means: Don’t try to do it. Don’t try to make an educated guess. Don’t even mark it to come back later. Just pick randomly, move on, and forget about this one forever.

Bail on these kinds of questions:

  • This is a big weakness of yours
  • You’ve read the problem and don’t understand what they’re asking or telling you—or you have no idea what to do with that information
  • You think you might know how to do it, but it would take you way too long (>4 minutes)
Now, let’s talk about the ones you do want to mark for a possible later return. First, be stingy. Don’t mark more than 2 questions in one section. You’re not going to have a ton of time left at the end; the last thing you want to do is spend a minute trying to figure out which of 4 marked problems you should actually return to…and then run out of time before you can try any of them.

When you mark a question for a possible later return, also put in a random answer right now. You may not actually make it back to this problem later, so it’s better to have a guess locked in, just in case. There’s no penalty for getting something wrong (vs. just not answering). And who knows—you might get lucky!

Mark these kinds of questions:

  • You know how to do this but it will take somewhat longer than average (3 to 4 minutes)
  • You’re thinking, “I know how to do this! I just did it last week! But I’m blanking right now. ☹”
For the first category, you just want to make sure that you don’t prevent yourself from finishing 2 questions at the end because you spent extra time on one long one earlier. Save that long one for last, just in case.

The second category is something that is in your brain somewhere, but you’re having trouble pulling up the memory right now. Sometimes, if we set the thing aside for 10 or 15 minutes, our brains will continue trying to figure it out subconsciously and then, when we look at it again, we’ll retrieve the memory: Oh, yeah! This is how to do this problem!

So if you run into one of those “But I know how to do this!” problems, don’t waste time trying to retrieve the memory right now. Let it percolate in the back of your brain while you do other stuff—then come back at the end (if you have time) to see whether you can pull up the memory now.

Time Management: Verbal and Quant
Verbal and Quant will work a lot like IR, with one twist thrown in: Your problems will be split into two separate panels and, when you move to the second panel, you can no longer go back to the first one.

That has two implications. First, don’t mark more than 1 question per panel.

Second, when you get to the end of the first panel, the test is going to ask whether you’re ready to move to the next one. Glance at the timer.

Recall that you have 30 minutes total, and each panel has 7 questions. So if you are “on time,” then you should have around 14 to 16 minutes left.

  • 14 to 16 minutes left? Keep going to the second panel; don’t go back to any marked questions from the first panel.
  • > 16 minutes left? Then you have a decision to make: Should you return to a marked question in the first panel or move on to the second panel?
Go glance at your 1 marked question. If it’s in the “I can do this but it’ll take 3 to 4 minutes” category, then decide whether you actually have the time to do it right now (and still feel confident that you can do it!).

If it’s in the “I’m blanking right now” category, re-read the problem. Has your subconscious memory figured it out? If so, solve. If you’re still thinking, “But I should know how…”—forget about it. Move on.

In Sum
As with any test, have a goal score. Talk to the schools to which you plan to apply to see whether they’ll tell you the kind of score that that school considers competitive.

Study IR, Verbal, and Quant relatively equally. You’ll likely spend a bit more time on your weaker areas (it takes us longer to improve our weaknesses), but don’t neglect your stronger areas—they need practice, too.

Do practice problems under timed conditions. At heart, the Executive Assessment is an executive reasoning / decision-making test, even while it tests you on math, logic, and grammar. As you do every day at work, you’re going to have to distinguish between good, mediocre, and bad opportunities and decide how to spend your limited time and mental energy accordingly.

Before you go in, know how you want to handle marking questions for later and bailing on questions during the test.

Good luck and happy studying! 

Now that you know how to study for the EA, sign up for our efficient and comprehensive Executive Assessment Masterclass!


Stacey Koprince is a Manhattan Prep instructor based in Montreal, Canada and Los Angeles, California. Stacey has been teaching the GMAT, GRE, and LSAT  for more than 15 years and is one of the most well-known instructors in the industry. Stacey loves to teach and is absolutely fascinated by standardized tests. Check out Stacey’s upcoming GMAT courses here.

The post How to Study for the Executive Assessment (Part 3) appeared first on GMAT.
This Blog post was imported into the forum automatically. We hope you found it helpful. Please use the Kudos button if you did, or please PM/DM me if you found it disruptive and I will take care of it. -BB
Manhattan GMAT Online Marketing Associate
Joined: 14 Nov 2013
Posts: 272
Own Kudos [?]: 51 [0]
Given Kudos: 0
Send PM
Stop Careless GMAT Quant Errors [#permalink]
Expert Reply
FROM Manhattan GMAT Blog: Stop Careless GMAT Quant Errors


Did you know that you can attend the first session of any of our online or in-person GMAT courses absolutely free? We’re not kidding! Check out our upcoming courses here.

Here’s a careless error that any of us might make:

x – 7 – 2x + 4 = 3x

-x – 3 = 3x

-3 = 2x

x = -1.5

Did you spot the error? If yes, give yourself a pat on the back and keep reading. If not, go back and review each step. This time, as you think through it, you can only use the terms added, subtracted, multiplied, and divided. On each line, identify which of those operations we used, and how we used it.

Why? Because there’s a common pattern in careless GMAT Quant errors. If you talk about the math you’re doing in vague, non-mathematical terms—saying things like “moved x to the other side” or “crossed off the terms”—your thinking becomes vague and non-mathematical as well. If you think and speak about GMAT Quant problems precisely and clearly, your work will be clearer and more precise. And clarity and precision keep you from making careless GMAT Quant errors.

Don’t say this: “Moved to the other side of the equation”

Say this instead: “Added to (or subtracted from) both sides”

Here’s the reasoning behind the careless error we looked at earlier. The second line looked like this:

-x – 3 = 3x

What next? You could think to yourself: “Let’s move the -x term to the right side of the equation.” That leads to the next line:

-3 = 2x

But that’s where the mistake happened! We weren’t supposed to subtract x from 3x; we were actually supposed to add it. But phrasing it as “move the term to the right side” doesn’t make that clear. If you stop and think about it, you’ll probably catch yourself and avoid making a mistake. But on the GMAT, you won’t have a lot of time to stop and check every step. You’ll be tired, and you might be anxious. Make things easier on yourself!

Instead of saying “move the term to the other side,” be precise about what you’re doing. Let’s start with that second line again:

-x – 3 = 3x

Next, think this: add x to both sides of the equation.

-x – 3 + x = 3x + x

-3 = 4x

By phrasing it like that, you totally avoid the careless GMAT Quant errors that come from “moving the term to the other side.” You don’t have to worry about whether to make x negative or positive—it doesn’t even enter into your thinking.

Don’t say this: “Crossed off the terms”

Say this instead: “Subtracted (or divided)”

Here’s another type of careless error. Imagine that you’re simplifying this equation:

2x² – 3y + 4 = y – 2 – 2x²

Cross off the 2x² on both sides to get -3y + 4 = y – 2, right? No. If you think in terms of “crossing off like terms,” you also need to double-check every single time to make sure that the signs match up, or else you’ll make a careless error. Instead, think this: subtract 2x² from both sides of the equation. That gives you the correct next step:

2x² – 3y + 4 – 2x²  = y – 2 – 2x² – 2x²

-3y + 4 = y – 2 – 4x²

This error shows up even more frequently when working with fractions. Check out this fraction, for example:



If you say to yourself “let’s cross off like terms,” you might quickly simplify the equation like this:



Don’t make that mistake! Instead, every time you simplify a fraction, remember that you’re dividing or multiplying the entire numerator and denominator by the same value. Here, if we tried to divide by x², we’d end up with a total mess:





Instead of doing that, identify something you can divide the entire numerator and denominator by evenly. For instance, you could divide both by 5.

Don’t say this: “Cross-multiplied”

Say this instead: “Found a common denominator,” “multiplied,” or “divided,” depending on what math you’re doing!

There’s nothing wrong with cross-multiplication. In fact, it’s a useful tool for one situation: when you want to determine which fraction is bigger. For instance, if you want to compare 3/11 and 4/17, you could cross-multiply:



This shows that 3/11 is bigger.

However, it’s easy to fall back on the term “cross-multiplying” to talk about situations it doesn’t actually apply to. This makes your work less clear. For example, if you’re just multiplying two fractions, you don’t want to accidentally cross-multiply! Instead, think to yourself: multiply the two numerators, and multiply the two denominators. If you’re adding two fractions together, don’t use the term “cross-multiplying”! Instead, think through the actual math you’re doing, which will include multiplication and addition. Here’s an example:



First, multiply both the numerator and denominator of the first fraction by 4:



Next, multiply both the numerator and denominator of the second fraction by 7:



Finally, you have a common denominator, so you can add the two fractions together. The answer is 15/28.

It’s okay to use the term “cross-multiply” when you want to figure out which of two fractions is larger. If you’re doing anything else, it’s safer to think about the specific math steps that you’re taking. That will keep you from accidentally cross-multiplying without thinking it through first.

In short, the language that you use when you think about math problems is important! A lot of careless GMAT Quant errors come from not being very clear and specific about what math you’re doing. If you use phrases like “crossed off the terms” as a shortcut, you might save a few seconds on some problems, but you’re also risking careless mistakes. If you challenge yourself to articulate exactly what math you’re doing at every step, you’ll not only avoid mistakes, you’ll also have a deeper understanding of how GMAT Quant works.

Don’t forget to follow us on Facebook, Twitter, and LinkedIn!

[b]Chelsey Cooley
 is a Manhattan Prep instructor based in Seattle, Washington.
 [/b]Chelsey always followed her heart when it came to her education. Luckily, her heart led her straight to the perfect background for GMAT and GRE teaching: she has undergraduate degrees in mathematics and history, a master’s degree in linguistics, a 790 on the GMAT, and a perfect 170/170 on the GRE. Check out Chelsey’s upcoming GRE prep offerings here.

The post Stop Careless GMAT Quant Errors appeared first on GMAT.
This Blog post was imported into the forum automatically. We hope you found it helpful. Please use the Kudos button if you did, or please PM/DM me if you found it disruptive and I will take care of it. -BB
Manhattan GMAT Online Marketing Associate
Joined: 14 Nov 2013
Posts: 272
Own Kudos [?]: 51 [0]
Given Kudos: 0
Send PM
To Keep or Cancel Your GMAT Scores? [#permalink]
Expert Reply
FROM Manhattan GMAT Blog: To Keep or Cancel Your GMAT Scores?


Did you know that you can attend the first session of any of our online or in-person GMAT courses absolutely free? We’re not kidding! Check out our upcoming courses here.

The GMAT offers various kinds of flexibility around your decision to keep or cancel your GMAT scores—but also some restrictions. It’s important to understand your options so that you make the best decision for you!

How does it work?
At the end of the test, you will be shown your scores (for everything except the essay) and you will then be asked whether you want to keep or cancel your GMAT scores. If you keep your scores, they’ll go on your official record. If you cancel them, they won’t; the school won’t see those scores, nor will the schools even know that you took the test that day.

Right at that moment, you’ll have 2 minutes to decide whether to keep or cancel your GMAT scores. Later, you can change your mind—but you’ll have to pay a fee to change the status of your scores. So let’s first talk about how to make the best decision during that first 2 minutes.

In short, you need to have an idea of what you’d want to do before you even walk in the testing room.

What do your schools want to see?
First, what kind of program do you want? MBA programs generally care only about your highest score. Other kinds of programs, such as Ph.D. programs, may look at all of your scores. So it’s important to find out how your schools are going to use the data.

If you are applying to an MBA program, you can assume that they don’t care if you take the test multiple times. They’re just going to use your best score and that’s that. If you are applying to a Ph.D. program or another type of Master’s program, ask the schools directly whether they care about multiple tests and, if so, how they use the multiple data points.

(By the way, for any school communication, I highly recommend attending one or more of the various MBA tours that travel around the country, giving you an opportunity to meet representatives from different schools. There are a bunch of different ones, some of which are tied to specific groups of people, such as women or other underrepresented groups. Ask your questions directly, make some connections, and get the ball rolling!)

What is your goal score?
Based on where you want to apply and how those schools use GMAT scores, you’ll come up with a goal score for yourself. Broadly speaking, you can classify the programs into one of three categories:

—Safety. I’m almost certain to be accepted to this school.

—Regular. I’ve got a good chance to get in, but it’s not a certainty.

—Reach. This school is a stretch, but hey, if I don’t even apply, I definitely won’t make it, right? So I’ll give it a shot.

Your GMAT goal should be above the average for your safety schools and at least at the average for your regular schools. You may not be above average for the reach schools, but you’d still ideally be within that school’s general range.

For the purposes of this discussion, let’s say that your goal score is a 650.

What is your minimum acceptable score?
Your ideal goal is 650, but let’s say that (based on your research) your minimum score is really a 620. You’d still feel comfortable applying to your schools with that score.

So, first, if you do hit a 620 or higher, you are not even going to think about canceling. You’re good to go!

What if you score a 610? Close enough. Keep it.

600?

580?

560…?

See where I’m going with this? ☺ At some point, the decision will switch to, “Nope, I’m going to cancel this one.” Where is that point?

Consider the Worst Possible Scenario
Your ideal goal is 650. Your minimum is 620. But what if you just can’t score above 590? You don’t want to take the test and score 590 and cancel, and then sign up again and get another 590 and cancel again, and then take it a third time and get a 550 because you’re so stressed out…and now you’ve taken the test three times and you have no score on your record at all.

The above scenario is even more likely for those who have really high goals. If someone really wants a 730 and keeps canceling 690 scores…that person might never make it to 730.

(You can reinstate your canceled scores at a later date—but you’re going to have to pay to do so. Let’s minimize your expenditure here.)

Know (More) about What the Schools Want
Remember how I said that MBA programs don’t really care if you take the test multiple times? For those programs, then, you don’t actually have to cancel anything. They don’t care. Just keep all your scores.

I know most students won’t be totally comfortable with this. I’m going to try to change your mind, though.

Anecdotally, we have heard that MBA programs, if anything, consider it a positive to see that you tried again. Let’s say that a school’s average is 650. You first score was a little under 650—say, 620 to 640. That’s probably good enough, but you decide to go for it again because you want to hit that average, if possible. This could play out in a couple of ways:

—You score 650+. Yay! You’re at/above the average for that school! Your hard work paid off.

—You increase your score a little but not all the way to the average. You are closer now, and you’ve signaled to the school that you were willing to try hard to succeed. They like to see that.

—You drop below your initial score. You still keep the score to signal to the school that you were willing to keep trying. Yeah, it did drop, but so what; you still have your original (higher) score locked in.

I would definitely keep the score in the first two scenarios. I also think it’s worth it to keep the score in the third scenario, but I would understand if a student didn’t feel comfortable doing so (particularly if you knew you would take the test a third time).

Final Advice: To Keep or Cancel Your GMAT Scores?
So all of the above leads me to this:

—If you’re applying for an MBA and you’re okay with my recommendation just to keep everything, then keep your score no matter what.

—If that idea makes you uneasy, then keep any score that’s within 100 points of your minimum goal score. If you want a minimum score of 650, keep any score of 550 or higher. (If your ideal score is 650 but your minimum is 620, keep anything at 520 or higher.)

Caveat: if your goal score is crazy high (e.g., you want a 780), keep anything within 150 points of your goal. I know, I know, a 630 isn’t anywhere near a 780. But less than 1% of the testing population hits a 780! That’s super ambitious. Be really happy if you get there, but don’t assume that anyone who just “studies enough” will get there.

I Canceled but Now I’m Thinking I Should Reinstate the Score… (or Vice Versa)
As of this writing, here are the details for canceling or reinstating a score after you leave the testing center. (Note that any details, especially pricing, could change in the future—so check mba.com to make sure that nothing has changed.)

If you keep your scores in the testing center but later decide that you want to cancel them, you have 72 hours to do so; after that, you cannot cancel your scores. You’ll have to pay a $25 fee.

If you cancel in the testing center but later decide that you want to reinstate your scores, you can do so as long as the scores are still valid (they expire after 5 years). This will cost you $50.

It costs less to cancel after the fact, but you have a time limit of 72 hours. If you’re just not sure what to do in the testing center, I would recommend keeping the scores, then using the next day or two to think about what to do (and ask others that you trust for their opinion). Then, if you do decide to cancel, you’ll only have to pay $25 to do so.

One Unusual Circumstance in Which You Actually Should Cancel
This last bit won’t apply to 99.9% of people taking the test, but just in case this happens to you, read on.

If you become ill or otherwise feel that you cannot finish while you are at the testing center, then a weird thing happens if you leave the test before getting to that “keep or cancel” screen at the very end. You won’t have any reported scores (since you didn’t finish) but the fact that you showed up to take the test that day will still show up on your official score report. It’s sort of an in-between case with an odd outcome.

So, if this happens to you, here’s what I recommend you do. If you have to leave the testing room (maybe you feel queasy and have to go to the bathroom), do so. Just let the test keep running. If you decide, when you get back, that you can’t keep going, then click through all of the remaining questions randomly to get yourself to the end of the test. On the Keep or Cancel screen, cancel your scores.

In Sum
Know what your goal scores (ideal and minimum) are.

Know what you want to do before you get into the testing room. (For example, tell yourself, “If I score 530 or higher, I’m keeping my score. If I score 520 or lower, I’m canceling.”)

If you just can’t decide at the end, keep the scores. Know that you’ll have 72 hours to change your mind and cancel instead. Get out of the testing room, clear your mind, decide what to do, and move ahead. 

Can’t get enough of Stacey’s GMAT mastery? Attend the first session of one of her upcoming GMAT courses absolutely free, no strings attached. Seriously.


Stacey Koprince is a Manhattan Prep instructor based in Montreal, Canada and Los Angeles, California. Stacey has been teaching the GMAT, GRE, and LSAT  for more than 15 years and is one of the most well-known instructors in the industry. Stacey loves to teach and is absolutely fascinated by standardized tests. Check out Stacey’s upcoming GMAT courses here.

The post To Keep or Cancel Your GMAT Scores? appeared first on GMAT.
This Blog post was imported into the forum automatically. We hope you found it helpful. Please use the Kudos button if you did, or please PM/DM me if you found it disruptive and I will take care of it. -BB
GMAT Club Bot
To Keep or Cancel Your GMAT Scores? [#permalink]
   1  ...  10   11   12   13   14  ...  21